User avatar
 
LSAT-Chang
Thanks Received: 38
Atticus Finch
Atticus Finch
 
Posts: 479
Joined: June 03rd, 2011
 
 
trophy
Most Thankful
trophy
First Responder
 

Q11 - Researcher: This fall I returned

by LSAT-Chang Sat Sep 24, 2011 6:42 pm

Hello!
I was wondering if an answer something like:

(F) It is unlikely that another researcher visited the same research site and tagged a new group of armadillos in an indistinguishable way as the researcher in question did.
(G) There are no new territories within a few hundred yards of the location of the armadillos' tagging last spring.

would have been correct as well. Any thoughts?

I was a little wary of (F), but I was thinking that if another researcher tagged a new group of armadillos in an indistinguishable way, then the researcher cannot use the sample he recaptured to make the conclusion since it could be that the ones he captured did move into rapidly new territories and the ones he captured are the ones that the new researcher tagged. Does this make any sense? I know this is kind of off-topic, but these two just came to mind when I was solving this problem, so I really wanted to hear some thoughts on other necessary assumptions.
 
shirando21
Thanks Received: 16
Atticus Finch
Atticus Finch
 
Posts: 280
Joined: July 18th, 2012
 
 
 

Re: Q11 - Researcher: This fall I returned

by shirando21 Thu Sep 13, 2012 3:21 pm

Premise:

This fall I returned to a research site to recover the armadillos I had tagged there the previous spring.

A large majority of the armadillos I recaptured were found within a few hundred yards of the location of their tagging last spring

Conclusion:

armadillos do not move rapidly into new territories.

(E)is required, because if you negate it, it will destroy the argument.
when you negate it, you get: a large majority of the recaptured armadillos moved to a new territory in the intervening summer and then moved back to the old territory by the fall. Then you can not get the conclusion based on the evidence/premises given in the argument.
 
claudia.minoiu
Thanks Received: 0
Vinny Gambini
Vinny Gambini
 
Posts: 6
Joined: April 09th, 2013
 
 
 

Re: Q11 - Researcher: This fall I returned

by claudia.minoiu Sat Apr 27, 2013 10:14 pm

What's wrong with A? Is it a sufficient condition?
 
sumukh09
Thanks Received: 139
Atticus Finch
Atticus Finch
 
Posts: 327
Joined: June 03rd, 2012
 
 
trophy
Most Thanked
trophy
First Responder
 

Re: Q11 - Researcher: This fall I returned

by sumukh09 Mon Apr 29, 2013 10:41 am

All A tells us it that when he tagged these armadillos a few were able to avoid being tagged when he visited the site last spring. This has nothing to do with the conclusion the researcher posits. So what if a few were able to avoid being tagged? If we negate this answer choice this wouldn't hurt the conclusion.
 
austindyoung
Thanks Received: 22
Elle Woods
Elle Woods
 
Posts: 75
Joined: July 05th, 2012
 
 
 

Re: Q11 - Researcher: This fall I returned

by austindyoung Wed May 22, 2013 6:20 pm

claudia.minoiu Wrote:What's wrong with A? Is it a sufficient condition?


I was between (A) and (E) and chose (E) because if false it destroys the argument.

(A) is tricky because I think it could possibly be the correct (or on the path to a correct) answer if this were a strengthen question. The conclusion is so broad, that it has the chance of inferring from a possible unrepresentative/too small a sample. So (A) tells us he caught more.

But this isn't necessary, in part I think because the conclusion is so broad that this doesn't need to be true. It's a matter of representation, unless of course the researcher tagged all armadillos.
User avatar
 
ManhattanPrepLSAT1
Thanks Received: 1909
Atticus Finch
Atticus Finch
 
Posts: 2851
Joined: October 07th, 2009
 
This post thanked 2 times.
 
 

Re: Q11 - Researcher: This fall I returned

by ManhattanPrepLSAT1 Fri May 24, 2013 3:47 pm

Great discussion so far! I thought I might just jump into this one too.

The argument concludes that armadillos do not move rapidly into new territory. Why? Because the majority of armadillos that the researcher recaptured, were found within a few hundred yards of where they were captured last spring.

The researcher may be correct in his/her conclusion, but he/she may also have missed something. Animals move around, and maybe the armadillos he found live mostly in one place, but during drought-ridden summer months migrate to better sources of water. If so, that would pose a serious threat to the researcher's conclusion. Answer choice (E) protects the argument from this possibility and so is required by the researcher's reasoning.

Incorrect Answers
(A) strengthens the argument, but doesn't need to be assumed. Sure it helps the argument if the researcher tagged all or nearly all of the armadillos in the area. But tagging just a few may be enough to track the movements of armadillos from that area.
(B) undermines the argument. This suggests the researcher is working limited information.
(C) is both too strong and out of scope. It needs to be true that at least some of the armadillos were not killed by predators, but we already know that some of them did - the researcher found them!
(D) strengthens the argument, but is not necessary. Even if some of the tags could be removed, does that suggest that the armadillos were moving into new areas?